Question and Answers Forum

All Questions      Topic List

Integration Questions

Previous in All Question      Next in All Question      

Previous in Integration      Next in Integration      

Question Number 39120 by math khazana by abdo last updated on 02/Jul/18

let A_n = ∫_1 ^n  (([(√(1+x^2 ))] −[x])/x^2 ) dx  (n integr ≥1)  1) calculate A_n   2) find lim_(n→+∞)  A_n

$${let}\:{A}_{{n}} =\:\int_{\mathrm{1}} ^{{n}} \:\frac{\left[\sqrt{\mathrm{1}+{x}^{\mathrm{2}} }\right]\:−\left[{x}\right]}{{x}^{\mathrm{2}} }\:{dx}\:\:\left({n}\:{integr}\:\geqslant\mathrm{1}\right) \\ $$$$\left.\mathrm{1}\right)\:{calculate}\:{A}_{{n}} \\ $$$$\left.\mathrm{2}\right)\:{find}\:{lim}_{{n}\rightarrow+\infty} \:{A}_{{n}} \: \\ $$

Commented by math khazana by abdo last updated on 04/Jul/18

we have A_n = Σ_(k=1) ^n  ∫_k ^(k+1)    (([(√(1+x^2 ))]−k)/x^2 )dx  but  k≤x<k+1 ⇒ k^2 ≤x^2 <(k+1)^2  ⇒  k^2  +1 ≤1+x^2 ≤1+ (k+1)^2  ⇒(√(1+k^2 )) ≤(√(1+x^2 )) <(√(1+(k+1)^2 ))  ⇒ (√(1+k^2 )) ≤(√(1+x^2 <)) (√(k^2  +2k+2))≤k+2 ⇒  k ≤ (√(1+x^2 <)) k+2  ⇒[(√(1+x^2 ))]=k or k+1   A_n =Σ_(k=1) ^n  ∫_k ^(k+1)  ((k+1−k)/x^2 )dx=Σ_(k=1) ^n  ∫_k ^(k+1)  (dx/x^2 )  =Σ_(k=1) ^n  [−(1/x)]_k ^(k+1)  = Σ_(k=1) ^n ( (1/k) −(1/(k+1)))  =1−(1/2) +(1/2) −(1/3) +....(1/n) −(1/(n+1)) =1−(1/(n+1)) ⇒  A_n = (n/(n+1))  2) its clear that  lim_(n→+∞)   A_n =1 .

$${we}\:{have}\:{A}_{{n}} =\:\sum_{{k}=\mathrm{1}} ^{{n}} \:\int_{{k}} ^{{k}+\mathrm{1}} \:\:\:\frac{\left[\sqrt{\mathrm{1}+{x}^{\mathrm{2}} }\right]−{k}}{{x}^{\mathrm{2}} }{dx}\:\:{but} \\ $$$${k}\leqslant{x}<{k}+\mathrm{1}\:\Rightarrow\:{k}^{\mathrm{2}} \leqslant{x}^{\mathrm{2}} <\left({k}+\mathrm{1}\right)^{\mathrm{2}} \:\Rightarrow \\ $$$${k}^{\mathrm{2}} \:+\mathrm{1}\:\leqslant\mathrm{1}+{x}^{\mathrm{2}} \leqslant\mathrm{1}+\:\left({k}+\mathrm{1}\right)^{\mathrm{2}} \:\Rightarrow\sqrt{\mathrm{1}+{k}^{\mathrm{2}} }\:\leqslant\sqrt{\mathrm{1}+{x}^{\mathrm{2}} }\:<\sqrt{\mathrm{1}+\left({k}+\mathrm{1}\right)^{\mathrm{2}} } \\ $$$$\Rightarrow\:\sqrt{\mathrm{1}+{k}^{\mathrm{2}} }\:\leqslant\sqrt{\mathrm{1}+{x}^{\mathrm{2}} <}\:\sqrt{{k}^{\mathrm{2}} \:+\mathrm{2}{k}+\mathrm{2}}\leqslant{k}+\mathrm{2}\:\Rightarrow \\ $$$${k}\:\leqslant\:\sqrt{\mathrm{1}+{x}^{\mathrm{2}} <}\:{k}+\mathrm{2}\:\:\Rightarrow\left[\sqrt{\mathrm{1}+{x}^{\mathrm{2}} }\right]={k}\:{or}\:{k}+\mathrm{1}\: \\ $$$${A}_{{n}} =\sum_{{k}=\mathrm{1}} ^{{n}} \:\int_{{k}} ^{{k}+\mathrm{1}} \:\frac{{k}+\mathrm{1}−{k}}{{x}^{\mathrm{2}} }{dx}=\sum_{{k}=\mathrm{1}} ^{{n}} \:\int_{{k}} ^{{k}+\mathrm{1}} \:\frac{{dx}}{{x}^{\mathrm{2}} } \\ $$$$=\sum_{{k}=\mathrm{1}} ^{{n}} \:\left[−\frac{\mathrm{1}}{{x}}\right]_{{k}} ^{{k}+\mathrm{1}} \:=\:\sum_{{k}=\mathrm{1}} ^{{n}} \left(\:\frac{\mathrm{1}}{{k}}\:−\frac{\mathrm{1}}{{k}+\mathrm{1}}\right) \\ $$$$=\mathrm{1}−\frac{\mathrm{1}}{\mathrm{2}}\:+\frac{\mathrm{1}}{\mathrm{2}}\:−\frac{\mathrm{1}}{\mathrm{3}}\:+....\frac{\mathrm{1}}{{n}}\:−\frac{\mathrm{1}}{{n}+\mathrm{1}}\:=\mathrm{1}−\frac{\mathrm{1}}{{n}+\mathrm{1}}\:\Rightarrow \\ $$$${A}_{{n}} =\:\frac{{n}}{{n}+\mathrm{1}} \\ $$$$\left.\mathrm{2}\right)\:{its}\:{clear}\:{that}\:\:{lim}_{{n}\rightarrow+\infty} \:\:{A}_{{n}} =\mathrm{1}\:. \\ $$$$ \\ $$

Answered by tanmay.chaudhury50@gmail.com last updated on 03/Jul/18

x      (√(1+x^2 ))      [(√(1+x^2 )) ]    [x_ ]    [(√(1+x^2 )) ]−[x]  1      (√2) =1.41               1        1                  0  1.5  (√(3.25)) =1.8     1      1               0  2        (√5)   =2.23            2        2                   0  2.5   (√(7.25))=2.69   2      2                0  3      (√(10)) =3.16         3      3                     0  4        (√(17)) = 4.12       4      4                   0     k      (√(1+k^2 ))  =k+△k   k     k            0  when  1 >△k>0   wait

$${x}\:\:\:\:\:\:\sqrt{\mathrm{1}+{x}^{\mathrm{2}} }\:\:\:\:\:\:\left[\sqrt{\mathrm{1}+{x}^{\mathrm{2}} }\:\right]\:\:\:\:\left[{x}_{} \right]\:\:\:\:\left[\sqrt{\mathrm{1}+{x}^{\mathrm{2}} }\:\right]−\left[{x}\right] \\ $$$$\mathrm{1}\:\:\:\:\:\:\sqrt{\mathrm{2}}\:=\mathrm{1}.\mathrm{41}\:\:\:\:\:\:\:\:\:\:\:\:\:\:\:\mathrm{1}\:\:\:\:\:\:\:\:\mathrm{1}\:\:\:\:\:\:\:\:\:\:\:\:\:\:\:\:\:\:\mathrm{0} \\ $$$$\mathrm{1}.\mathrm{5}\:\:\sqrt{\mathrm{3}.\mathrm{25}}\:=\mathrm{1}.\mathrm{8}\:\:\:\:\:\mathrm{1}\:\:\:\:\:\:\mathrm{1}\:\:\:\:\:\:\:\:\:\:\:\:\:\:\:\mathrm{0} \\ $$$$\mathrm{2}\:\:\:\:\:\:\:\:\sqrt{\mathrm{5}}\:\:\:=\mathrm{2}.\mathrm{23}\:\:\:\:\:\:\:\:\:\:\:\:\mathrm{2}\:\:\:\:\:\:\:\:\mathrm{2}\:\:\:\:\:\:\:\:\:\:\:\:\:\:\:\:\:\:\:\mathrm{0} \\ $$$$\mathrm{2}.\mathrm{5}\:\:\:\sqrt{\mathrm{7}.\mathrm{25}}=\mathrm{2}.\mathrm{69}\:\:\:\mathrm{2}\:\:\:\:\:\:\mathrm{2}\:\:\:\:\:\:\:\:\:\:\:\:\:\:\:\:\mathrm{0} \\ $$$$\mathrm{3}\:\:\:\:\:\:\sqrt{\mathrm{10}}\:=\mathrm{3}.\mathrm{16}\:\:\:\:\:\:\:\:\:\mathrm{3}\:\:\:\:\:\:\mathrm{3}\:\:\:\:\:\:\:\:\:\:\:\:\:\:\:\:\:\:\:\:\:\mathrm{0} \\ $$$$\mathrm{4}\:\:\:\:\:\:\:\:\sqrt{\mathrm{17}}\:=\:\mathrm{4}.\mathrm{12}\:\:\:\:\:\:\:\mathrm{4}\:\:\:\:\:\:\mathrm{4}\:\:\:\:\:\:\:\:\:\:\:\:\:\:\:\:\:\:\:\mathrm{0}\: \\ $$$$ \\ $$$${k}\:\:\:\:\:\:\sqrt{\mathrm{1}+{k}^{\mathrm{2}} }\:\:={k}+\bigtriangleup{k}\:\:\:{k}\:\:\:\:\:{k}\:\:\:\:\:\:\:\:\:\:\:\:\mathrm{0} \\ $$$${when}\:\:\mathrm{1}\:>\bigtriangleup{k}>\mathrm{0}\: \\ $$$${wait} \\ $$$$ \\ $$

Commented by abdo.msup.com last updated on 03/Jul/18

but you know that the train dont wait...

$${but}\:{you}\:{know}\:{that}\:{the}\:{train}\:{dont}\:{wait}... \\ $$

Commented by tanmay.chaudhury50@gmail.com last updated on 03/Jul/18

Commented by tanmay.chaudhury50@gmail.com last updated on 03/Jul/18

f(x)=[(√(1+x^2 )) ]−[x]  from graph it is clear that f(x)=[(√(1+x^2 )) ]−[x]  is zero when n>x≥1  so the value of intregal  is zero...pls check

$${f}\left({x}\right)=\left[\sqrt{\mathrm{1}+{x}^{\mathrm{2}} }\:\right]−\left[{x}\right] \\ $$$${from}\:{graph}\:{it}\:{is}\:{clear}\:{that}\:{f}\left({x}\right)=\left[\sqrt{\mathrm{1}+{x}^{\mathrm{2}} }\:\right]−\left[{x}\right] \\ $$$${is}\:{zero}\:{when}\:{n}>{x}\geqslant\mathrm{1} \\ $$$${so}\:{the}\:{value}\:{of}\:{intregal}\:\:{is}\:{zero}...{pls}\:{check} \\ $$

Commented by math khazana by abdo last updated on 03/Jul/18

miracylous graph for this function it s like  a rail ways...

$${miracylous}\:{graph}\:{for}\:{this}\:{function}\:{it}\:{s}\:{like} \\ $$$${a}\:{rail}\:{ways}... \\ $$

Terms of Service

Privacy Policy

Contact: info@tinkutara.com